LSAT and Law School Admissions Forum

Get expert LSAT preparation and law school admissions advice from PowerScore Test Preparation.

 ltowns1
  • Posts: 61
  • Joined: May 16, 2017
|
#43689
Could someone help me with why (D) is wrong? I'm always confused about what I should read in the stimulus when it says things like " the second causes the first and the first causes the second" lol.
 James Finch
PowerScore Staff
  • PowerScore Staff
  • Posts: 943
  • Joined: Sep 06, 2017
|
#43715
Hi LTowns,

What (D) means is that the stimulus is reversing cause and effect, which is not the case here. Instead we see causation and correlation confused: that because disease X causes one symptom, which typically correlates with another medical issue, that the underlying disease must also be a cause of the correlated issue, by implying high angiotensinogen levels cause high blood pressure.

Hope this clears things up!
 ltowns1
  • Posts: 61
  • Joined: May 16, 2017
|
#43738
Okay thanks
 coolbeans747
  • Posts: 7
  • Joined: May 16, 2018
|
#46329
In the correct answer choice C, the “two phenomena being correlated” are a persons angiotensinogen levels and blood pressure, correct?

Also, why is C a better choice than B?

Thanks
 Alex Bodaken
PowerScore Staff
  • PowerScore Staff
  • Posts: 136
  • Joined: Feb 21, 2018
|
#46352
coolbeans747,

Thanks for the question! Spot on - the two things being correlated are angiotensinogen and blood pressure.

As to why (C) is a better answer than (B): the weakness of (B) as an answer is that we are provided no evidence of "other factors that may fully counteract" the effect that disease X has on blood pressure. Because there is no evidence provided of these factors, it isn't a valid criticism to say that the author overlooks this possibility. Answer choice (C), by contrast, speaks to a flaw that is inherent in causal reasoning, which is that we can never be 100% certain that one thing causes another just because another thing happens after it. Therefore, it is the better answer choice.

Hope that helps!
Alex
 haganskl
  • Posts: 43
  • Joined: May 30, 2019
|
#76600
Hello.
I have a question about the stim. So did the doctor, the freaking DOCTOR LOL, rationalize the causal relationship like this?

C: Disease X Effect: Long A word

C: Long A word Effect: High blood pressure

Therefore:
C: Disease X Effect: High blood pressure

?

I illicitly chose answer choice B. I read above why its wrong. I immediately got rid of C because I didn’t see the connection, even though it’s SO clear. Smh

Thanks in advance for taking my question.
 Adam Tyson
PowerScore Staff
  • PowerScore Staff
  • Posts: 5153
  • Joined: Apr 14, 2011
|
#76992
The Doctor did indeed make a bad causal argument, haganski! But the evidence was not that A causes high blood pressure. The evidence was that higher A levels are correlated with higher BP. If we knew that disease X caused higher levels of A, and if we also knew that higher levels of A cause higher BP, then it would be valid to conclude that Disease X causes higher BP. The problem is that we don't know that A and BP have a causal relationship, but only that they are correlated.
 haganskl
  • Posts: 43
  • Joined: May 30, 2019
|
#77005
Got it! Thank you.

Get the most out of your LSAT Prep Plus subscription.

Analyze and track your performance with our Testing and Analytics Package.